PT9.S4.Q23

I diagramed the stimulus as follow:
.
Conflicting behavior organization --> More pain and distress
Conflicting behavior organization --> Animals resist --> Less efficient
.
My question is I don't understand why (E) is correct. I feel like (E) negates the sufficient condition to state the answer, which is an error.
.
Somebody please help! Thank you!!!!!

Comments

  • SamiSami Live Member Sage 7Sage Tutor
    10789 karma

    I think the issue with doing this stimulus by diagramming it conditionally is that this stimulus is not meant to be conditional but comparative in nature. You are not just considering conflicting behavioral organization but also comparing it to non conflicting ones.

    1. Pain and distress
      Current farm practices > practices that conform more closely to animals behavioral tendencies
      This means current farm practices cause more pain and distress than the other practice. ( it doesn't mean the other one does not, just it causes more)

    2. less efficient
      Current farm practices > practices that conform more closely to animals behavioral tendencies.
      So current farm practices again are less efficient than this other practice that matches animals behavioral tendencies better.

    If you were a farmer wouldn't this information make you consider that if you made changes to your farm practices and made it less painful and distressful then its possible that your farm as a result will be more efficient?

    I know based on this information I would like to make my farm practices more closely conform to animals behavioral tendencies.

    Answer choice E is perfect in a sense that its not strong, it uses the word "can result" which leaves open the possibility that switching it might not work which we can conclude appropriately.

  • hihihi9993hihihi9993 Member
    edited October 2017 342 karma

    @Sami Thank you! "Can" can't be represented as conditional diagram because it it is not absolute! Got it!

  • SamiSami Live Member Sage 7Sage Tutor
    10789 karma

    @d931n027h said:
    @Sami Thank you! "Can" can't be represented as conditional diagram because it it is not absolute! Got it!

    Yes, :) that and the stimulus is dealing with comparing two things so you don't want to diagram it as conditional because that's now how the argument is structured and if you do that you will miss out on some things the stimulus is saying.

Sign In or Register to comment.